Author Topic: Day Section's Quiz - Problem 1  (Read 5071 times)

Sabrina (Man) Luo

  • Jr. Member
  • **
  • Posts: 9
  • Karma: 4
    • View Profile
Day Section's Quiz - Problem 1
« on: April 03, 2013, 09:11:21 AM »
(1) For the system
                           \begin{equation*}
\left\{\begin{aligned}
&dx/dt=y+x(1-x^2-y^2),\\
&dy/dt=-x+y(1-x^2-y^2)
\end{aligned}
\right.\end{equation*}
 determine all critical points, linearize around each critical point, and determine what conclusion can be made about the nonlinear system at each critical point based on the linearization. Draw a phase portrait for the nonlinear system.

« Last Edit: April 03, 2013, 10:15:57 AM by Victor Ivrii »

Victor Ivrii

  • Administrator
  • Elder Member
  • *****
  • Posts: 2607
  • Karma: 0
    • View Profile
    • Personal website of Victor Ivrii
Re: Day Section's Quiz - Problem 1
« Reply #1 on: April 03, 2013, 09:29:47 AM »
Sabrina, thanks for posting problems (I "LaTeXed" them). Also you should post different problems in different topics.

Pictures have too low resolution to be read. However in Problem 1 at least one can see a correctly sketched equilibrium point.

PS: After you sent to me actual jpg files (each more than 1MB I removed colour but result is not as good as if the original scan was B/W). It should be < 100 kb per page

Bonus question to this problem: are there any closed trajectories? Hint: use polar coordinates.
Actually this bonus question deals with a notion introduced in more advanced section of Chapter 9.
« Last Edit: April 03, 2013, 10:22:00 AM by Victor Ivrii »

Alexander Jankowski

  • Full Member
  • ***
  • Posts: 23
  • Karma: 19
    • View Profile
Re: Day Section's Quiz - Problem 1
« Reply #2 on: April 03, 2013, 10:37:32 AM »
The critical points can be determined by solving the equations
 \begin{equation*}
 0 = y + x(1 - x^2 - y^2), \qquad 0 = -x + y(1 - x^2 - y^2).
 \end{equation*}
It turns out that the only critical point is $(0,0)$. Now, let
 \begin{equation*}
 F(x,y) = y + x(1 - x^2 - y^2), \qquad G(x,y) = -x + y(1 - x^2 - y^2).
 \end{equation*}
The Jacobian matrix of these functions is
 \begin{equation*}
 J = \left( \begin{array}{cc} 1 - 3x^2 - y^2 & 1 - 2xy \\ -1 - 2xy & 1 - x^2 - 3y^2 \end{array} \right).
 \end{equation*}
Evaluating it at $(0,0)$, we find that
 \begin{equation*}
 J = \left( \begin{array}{rr} 1 & 1 \\ -1 & 1 \end{array} \right).
 \end{equation*}
The linear system at $(0,0)$ is then
 \begin{equation*}
 \frac{d}{dt} \left( \begin{array}{c} x \\ y \end{array} \right) = \left( \begin{array}{rr} 1 & 1 \\ -1 & 1 \end{array} \right)
 \left( \begin{array}{c} x \\ y \end{array} \right).
 \end{equation*}
The eigenvalues of the coefficient matrix are
 \begin{equation*}
 \text{det} \left( \begin{array}{cc} 1 - \lambda & 1 \\ -1 & 1 - \lambda \end{array} \right) = \lambda^2 - 2\lambda + 2 = 0
 \Longleftrightarrow \lambda = 1 \pm i.
 \end{equation*}
The eigenvalues are complex with positive real part. Therefore, $(0,0)$ is an unstable spiral point. We can conclude that the non-linear system behaves similarly. To determine the orientation of the spiral, we use the vector $(x,y)^T = (0,1)$ in the equation of the system to find that
 \begin{equation*}
 \frac{d}{dt} \left( \begin{array}{c} x \\ y \end{array} \right) = \left( \begin{array}{rr} 1 & 1 \\ -1 & 1 \end{array} \right)
 \left( \begin{array}{c} 0 \\ 1 \end{array} \right) = \left( \begin{array}{c} 1 \\ 1 \end{array} \right).
 \end{equation*}
This tells us that the spiral is oriented clockwise. A stream plot of the system is attached. Note that there is a closed trajectory $x^2 + y^2 = 1$ that exhibits orbital stability. It is the limit cycle of the system and is approached by all trajectories as $ t\rightarrow \infty$. This can be shown rigorously by applying the same procedure that is given in Example 1 of Section 9.7--in fact, the polar system of equations that must be solved is the exact same.
« Last Edit: April 03, 2013, 10:40:33 AM by Alexander Jankowski »

Victor Ivrii

  • Administrator
  • Elder Member
  • *****
  • Posts: 2607
  • Karma: 0
    • View Profile
    • Personal website of Victor Ivrii
Re: Day Section's Quiz - Problem 1
« Reply #3 on: April 03, 2013, 11:04:42 AM »
Yes. this is a stable limit cycle. In fact it is stable from both inside and outside.

« Last Edit: April 03, 2013, 01:14:17 PM by Victor Ivrii »

Iven Poon

  • Newbie
  • *
  • Posts: 4
  • Karma: 2
    • View Profile
Re: Day Section's Quiz - Problem 1
« Reply #4 on: April 03, 2013, 02:16:21 PM »
I already written my solution down on Monday, and I planned to post on Thursday when Professor Ivrii uploaded the questions...

Anyway, I would still like to share how I did this question.


Victor Ivrii

  • Administrator
  • Elder Member
  • *****
  • Posts: 2607
  • Karma: 0
    • View Profile
    • Personal website of Victor Ivrii
Re: Day Section's Quiz - Problem 1
« Reply #5 on: April 03, 2013, 04:30:52 PM »
I already written my solution down on Monday, and I planned to post on Thursday when Professor Ivrii uploaded the questions...

Anyway, I would still like to share how I did this question.

Not me—Sabrina. I just improved her post and split it into 2